You are on page 1of 8
Control Systems ME Ph.D. Qualifying Examination Syllabus Exam content (based on material covered in the following courses): ‘ME 360, Signal Processing, Instrumentation and Control ‘ME 460, Industrial Controls Systems or ECE 486 Exam Topi 1, Basics + Fourier Transform + Signal decomposition using Fourier Transforms + Laplace Transform + Solutions of ODEs using Transforms + Convolution + Power Spectral Density 2. Modeling of Physical Systems Slements of Physical Systems and Block Diagrams + Frequency Response and Transfer Functions + System Identification from Input Output Data + Linearization + System Types (Type 1, Type 2) and Steady State Tracking + State-space representation of systems 3. Time Domain Analysis + Transient Response( st & 2nd Order systems) + Impulse Response + Foreed Response/Steady State Response + Phase Lead, Phase Lag 4, Basic Control Actions + Proportional Control + Integral Control * PID Control Stability + Poles/Eigenvalues + Bounded Input Bounded Output Stability + Stability Tests(Routh-Hurwitz Criteria) 6. Root Locus Techniques * Construction of a Root Locus + Analysis and Design (Lead, Lag, Lead-Lag) + Bode Analysis and Design + Nyquist Analysis and Design + Stability: Gain Margin/Phase Margin + Sensitivity to Disturbances and Model Uncertainty + Lead, Lag, Lead-Lag Design Reference Textbooks: 1) Ashok Ambardar, Analog and Digital Signal Processing, 2" Edition, Brooks/Cole Publishing, 1999 2) I. Cochin, W. Cadwallender, “Analysis and Design of Dynamical Systems,” 3" edition, Addision-Wesley, (1997) 3) K. Ogata, Modern Control Engineering, 4th Edition, Prentice Hall, 2002 4) G.F. Franklin, J. D. Powell, A, Emami-Naeini, Feedback Control of Dynamic Systems, 4th Edition, Prentice Hall, 2002 5) B, Kuo, Automatic Control Systems, 6th Edition, Prentice Hall, 1991 6) K. Astrom, R. Murray, Feedback Systems: An introduction for scientists and engineers, ISBN-13: 978-0691 135762, 2008 QUALIFYING EXAMINATION FOR CONTROL SYSTEMS Department of Mechanical and Industrial Engineering University of Illinois at Urbana-Champaign Wednesday, August _, | 9:00 AM — 12:00 PM | IMPORTANT EXAMINATION INFORMATION 1, Identify your examination and work with your University Identification Number (UIN, I-Card number in blue beginning with 65) on each page. DO NOT ENTER YOUR NAME ANYWHERE IN THE EXAMINATION. 2. Choose 3 out of the 4 problems. | 3. Each problem counts 10 points, 4. Start each problem in a new examination booklet and write on only the right-hand side (front side) of each sheet. 5. Hand in this problem package with your exam booklets. ea on a a ee ee ee ee SAMPLE EXAM FOR CONTROLS ME PHD Qualifying Exam Problem 1 Consider a waveforin given by x(t)=sin(400xt+7/6) - cos(600xt+n/8). What is the fundamental frequency of the waveform? Is x(t) decomposed into Fourier series? Which kind? Decompose x(t) into trigonometric, polar, and exponential Fourier series. Draw its magnitude and phase spectra. Does x(t) have Fourier Transform? Which kind? If it does, give it analytically and draw power spectral density of x(0. Problem Consider the block diagram system shown below: Ge ios + + G Ge | Gs - + (@) Determine the transfer function from R(S) to Y(s) (b) Suppose all the transfer function blocks G; (i¢[1,6]) are stable. Can you tell anything about the overall system stability without knowing more? Problem 2: Part (b) Real Axis Problem 3 Consider the third order system in unity feedback (negative feedback) given below + +Y(s) R6) GHys? 4s + 8) ‘The Nyquist Plot for this controlled system with K=10 is given below, . Nyquist Diagram 1. 08- : J i id | Imaginary Axis Ea 0.2040608 1 1214 ea Ax ° Problem 3 continued: (a) Determine the Gain Margin and Phase Margin for the system with gain K= 10. ‘An approximation based on graphical reading will be sufficient. However, you must show the reasoning/calculation behind your answer. (b) Draw the Root Locus for this system (K=0 to infinity) including any applicable: (1) Open Loop Poles and Zeros @) — Asymptotes. @) —_ Breakaway/Break-in points (4) Departure and Arrival angles | ©) _ je axis cross over points (©) What is the range of values for K that we can expect the system to remain stable for? | | Problem 4 Consider the unity feedback system (negative feedback) with the plant given by a static gain (= 5) and controlled by the P and PI controller in the forward path. Shown in the figure below is the PI version of the controller. Ye) RG) (@) Analytically compute the output of the system for a unit step input for both a P controller and a Pl controller. You should give two separate answers for y(t). + For the P controller, assume K, ~ 50 (and obviously Ki = 0). * For Pl controller, assume K, = 50 and Ky= 4, (b) In addition to analytically computing the system output, you must also sketch (to the best of your ability) the output for both of the control systems. + Itmay be very useful to put graphs of both systems on the same plot. (©) Indicate the difference between the closed loop steady state errors for these two controllers, if any, and support your explanation analytically. (@ Give analytical expressions for, and draw Bode Diagrams of, the closed loop for each controller. «Need magnitude and phase diagrams (© Analyze and explain the qualitative frequency domain (magnitude and phase) actions of each controller and indicate the type of filtering action each closed loop performs.

You might also like